subject
Mathematics, 13.08.2019 05:10 Gipson

Aconvenience store manager noticed that a cooler which had been stocked with only a cola product and an iced-tea product had 15 colas left at the end of the day but only 3 iced-tea beverages. as a result, the manager reasoned that he should increase the amount of iced tea and decrease the amount of cola he ordered from the distributor. which of the following, if true, would most strengthen the manager's rationale for ordering more iced tea and less cola? a) the cooler in question is the only place in the store where the cola and iced tea beverages are stocked. b)on that day, a month-long $1,000,000 sweepstakes began, with prizes awarded via the bottlecaps on the iced tea beverage. c)at the beginning of the day, the cooler was stocked with at least as many of the iced tea beverages as of the cola beverages. d)on the subsequent day, the remaining three iced tea beverages all sold within the first hour after the store opened. e)during that week, a special "buy one, get one free" sale was in effect for the cola beverage.

ansver
Answers: 2

Another question on Mathematics

question
Mathematics, 21.06.2019 14:40
Simplify.7x + 3y - 2 + 6x + 1 + y squared 2 you! ; )
Answers: 1
question
Mathematics, 21.06.2019 18:30
Is the square root of 4 plus the square root of 16 rational?
Answers: 2
question
Mathematics, 21.06.2019 18:30
Deshawn uses 3/4 of a box of rice to cook dinner the portion he uses weighs 12 ounces right and solve an equation to find the weight of the full box of rice
Answers: 1
question
Mathematics, 21.06.2019 20:50
Write the equation in logarithmic form e^x=403.429
Answers: 1
You know the right answer?
Aconvenience store manager noticed that a cooler which had been stocked with only a cola product and...
Questions
question
Mathematics, 13.04.2021 03:00
Questions on the website: 13722359